LSAT 119 – Section 4 – Question 17
You need a full course to see this video. Enroll now and get started in less than a minute.
Target time: 1:30
This is question data from the 7Sage LSAT Scorer. You can score your LSATs, track your results, and analyze your performance with pretty charts and vital statistics - all with a Free Account ← sign up in less than 10 seconds
Question QuickView |
Type | Tags | Answer Choices |
Curve | Question Difficulty |
Psg/Game/S Difficulty |
Explanation |
---|---|---|---|---|---|---|---|
PT119 S4 Q17 |
+LR
+Exp
| Weaken +Weak Causal Reasoning +CausR | A
5%
155
B
2%
153
C
7%
157
D
15%
158
E
71%
163
|
140 151 162 |
+Medium | 145.1 +SubsectionEasier |
Researcher: Hard water contains more calcium and magnesium than soft water contains. Thus, those who drink mostly soft water incur an increased risk of heart disease, stroke, and hypertension, for people being treated for these conditions tend to have lower levels of magnesium in their blood.
Summarize Argument: Phenomenon-Hypothesis
The researcher hypothesizes that people who drink mostly soft water have a higher risk of heart disease, stroke, and hypertension than people who drink mostly hard water. This is because hard water has more magnesium than soft water. The researcher’s hypothesis is based on the phenomenon that people being treated for the aforementioned conditions having lower levels of magnesium in their blood.
Notable Assumptions
The researcher assumes that the difference in hard and soft water’s magnesium levels is not negligible. The researcher also assumes that lower levels of magnesium (at least in part) caused people to develop heart disease, stroke, and hypertension, as opposed to the low magnesium levels being an effect of these conditions.
A
Magnesium deficiency is not uncommon, even in relatively prosperous countries with an otherwise generally adequate diet.
This does not affect the argument. The researcher does not discuss magnesium deficiencies: he focuses on the relative magnesium levels in hard and soft water, and the low magnesium levels in the blood of people receiving treatment. We cannot conclude that they are deficient.
B
Magnesium is needed to prevent sodium from increasing blood pressure.
This does not affect the argument. The role of magnesium in maintaining blood pressure is not relevant to the argument.
C
As people age, their ability to metabolize magnesium deteriorates.
This does not affect the argument. We don’t know about the ages of the patients being treated or how age affects one’s likeliness to develop one of the mentioned conditions.
D
The ingestion of magnesium supplements inhibits the effectiveness of many medicines used to treat high blood pressure and heart disease.
This does not affect the argument. (D) suggests those being treated for these conditions cannot take supplements to increase their low magnesium levels. This is compatible with the author’s assumption that lower levels of magnesium increase one’s risk of the conditions described.
E
Compounds commonly used to treat hypertension and heart disease diminish the body’s capacity to absorb and retain magnesium.
This weakens the argument. It exploits the researcher’s assumption that lower magnesium levels caused people to need treatment for the conditions described. (E) says the opposite: that the lower magnesium levels are a result of receiving treatment.
Take PrepTest
Review Results
LSAT PrepTest 119 Explanations
Section 1 - Reading Comprehension
- Passage 1 – Passage
- Passage 1 – Questions
- Passage 2 – Passage
- Passage 2 – Questions
- Passage 3 – Passage
- Passage 3 – Questions
- Passage 4 – Passage
- Passage 4 – Questions
Section 2 - Logical Reasoning
- Question 01
- Question 02
- Question 03
- Question 04
- Question 05
- Question 06
- Question 07
- Question 08
- Question 09
- Question 10
- Question 11
- Question 12
- Question 13
- Question 14
- Question 15
- Question 16
- Question 17
- Question 18
- Question 19
- Question 20
- Question 21
- Question 22
- Question 23
- Question 24
- Question 25
Section 3 - Logical Reasoning
- Question 01
- Question 02
- Question 03
- Question 04
- Question 05
- Question 06
- Question 07
- Question 08
- Question 09
- Question 10
- Question 11
- Question 12
- Question 13
- Question 14
- Question 15
- Question 16
- Question 17
- Question 18
- Question 19
- Question 20
- Question 21
- Question 22
- Question 23
- Question 24
- Question 25
- Question 26
Leave a Reply
You must be logged in to post a comment. You can get a free account here.